Problemas

Esta es nuestra colección de problemas. Los hemos clasificados por tema, dificultad y tipo de concurso. No dudes en escribir comentarios con tus soluciones o con cualquier duda sobre el problema.
También puedes compartirnos alguno de tus problemas favoritos:
Problema

Múltiplos de un primo escritos con puros unos

Enviado por jmd el 19 de Diciembre de 2011 - 21:33.

 Demostrar que para todo número primo $p$ distinto de 2 y de 5, existen infinitos múltiplos de $p$ de la forma 1111...1 (escrito sólo con unos).

Problema

Desigualdad con inradio y circunradio

Enviado por jmd el 19 de Diciembre de 2011 - 21:32.

Justificar razonadamente que, en cualquier triángulo, el diámetro de la circunferencia inscrita no es mayor que el radio de la circunferencia circunscrita.

Problema

Triángulo aritmético

Enviado por jmd el 19 de Diciembre de 2011 - 21:30.

Sea dado el triángulo aritmético

0 1 2 3 4 ............. 1991 1992 1993
 1 3 5 7...................... 3983 3985
  4 8 12............................. 7968
...
(donde cada número es la suma de los dos que tiene encima, cada fila tiene un número menos y en la última sólo hay un número). Demostrar que el último número es múltiplo de 1993.

Problema

Pichoneras de nacionalidad, edad y sexo

Enviado por jmd el 19 de Diciembre de 2011 - 21:27.

En una reunión hay 201 personas de 5 nacionalidades diferentes. Se sabe que, en cada grupo de 6, al menos dos tienen la misma edad. Demostrar que hay al menos 5 personas del mismo país, de la misma edad y del mismo sexo.

Problema

Segmentos formados por n puntos

Enviado por jmd el 10 de Diciembre de 2011 - 21:29.

Se tienen $n$ puntos distintos $A_1, A_2,\ldots,A_n$ en el plano y a cada punto $A_i$ se ha asignado un número real $\lambda$ distinto de cero, de manera que $\overline{A_iA_j}^2=\lambda_i+\lambda_j$, para todos los $i,j,i\neq j$
 Demuestre que
(a) $n\leq 4$
(b) Si $n = 4$, entonces $\frac{1}{\lambda_1}+\frac{1}{\lambda_2}+\frac{1}{\lambda_3}+\frac{1}{\lambda_4}=0$

Problema

Coloreo de triángulos con fichas

Enviado por jmd el 10 de Diciembre de 2011 - 21:17.

Tres fichas $A, B, C$ están situadas una en cada vértice de un triángulo equilátero de lado $n$. Se ha dividido el triángulo en triangulitos equiláteros de lado 1, tal como muestra la figura en el caso $n = 3$.

Inicialmente todas las líneas de la figura están pintadas de azul. Las fichas se desplazan por las líneas, pintando de rojo su trayectoria, de acuerdo con las dos reglas siguientes:

Problema

Suma de fracciones 1/ab

Enviado por jmd el 10 de Diciembre de 2011 - 21:16.

Dado un número natural $n\geq 2$ considere todas las fracciones de la forma $1/ab$, donde $a$ y $b$ son números naturales, primos entre sí y tales que $$a < b \leq n$$ $$a + b \gt n$$ Demuestre que para cada $n$, la suma de estas fracciones es 1/2.

 

Problema

Método para distribuir ceros y unos en un tablero

Enviado por jmd el 10 de Diciembre de 2011 - 21:13.

Tenemos un tablero cuadriculado de $k^2 - k + 1$ filas y $k^2 - k + 1$ columnas, donde $k = p + 1$ y $p$ es un número primo. Para cada primo $p$, dé un método para distribuir números entre 0 y 1, un número en cada casilla del tablero, de modo que en cada fila haya exactamente $k$ números $0$ en cada columna haya exactamente $k$ números $0$ y además no haya ningún rectángulo de lados paralelos a los lados del tablero con números 0 en sus cuatro vértices.

 

Problema

Punto medio de la mediana

Enviado por jmd el 10 de Diciembre de 2011 - 21:11.

 Sea $M$ el punto medio de la mediana $AD$ del triángulo $ABC$ ($D$ pertenece al lado $BC$). La recta $BM$ corta al lado $AC$ en el punto $N$. Demuestre que $AB$ es tangente a la circunferencia circunscrita al triángulo $NBC$ si, y sólo si, se verifica la igualdad $$\frac{BM}{MN}=\left(\frac{BC}{BN}\right)^2$$

Problema

Cubo formado por 1996 cubos

Enviado por jmd el 10 de Diciembre de 2011 - 21:09.

Sea $n$ un número natural. Un cubo de arista $n$ puede ser dividido en $1996$ cubos cuyas aristas son también números naturales. Determine el menor valor posible de $n$.

Problema

Grado de repulsión de una función circular

Enviado por jmd el 10 de Diciembre de 2011 - 15:44.

Una función $f: N \mapsto N$ es circular si para cada $p$ en $N$ existe $n$ en $N$ con $n\leq p$ tal que:
$$\underbrace{f^n(p) = f(f(\ldots f(p) \ldots )))}_{n veces}=p$$
La función $f$ tiene grado de repulsión $k$, $0 < k < 1$, si para cada $p$ en $N$, $f^i(p) \neq p$ para $i\leq [k\cdot p]$. Determine el mayor grado de repulsión que puede tener una función circular. Nota: $[x]$ indica el mayor entero menor o igual que $x$.

 

Problema

... y se forma un trapecio isósceles...

Enviado por jmd el 10 de Diciembre de 2011 - 15:38.

La circunferencia inscrita en el triángulo $ABC$ es tangente a $BC, CA$ y $AB$ en $D, E$ y $F$, respectivamente. Suponga que dicha circunferencia corta de nuevo a $AD$ en su punto medio $X$, es decir, $AX = XD$. Las rectas $XB$ y $XC$ cortan de nuevo a la circunferencia inscrita en $Y$ y en $Z$, respectivamente. Demuestre que $EY = FZ$.

Problema

Dominio eficiente de un tablero

Enviado por jmd el 10 de Diciembre de 2011 - 15:36.

En un tablero de $m\times m$ casillas se colocan fichas. Cada ficha colocada en el tablero "domina" todas las casillas de la fila (--), la columna (|) y la diagonal (\), a la que pertenece. Determine el menor número de fichas que deben colocarse para que queden "dominadas" todas las casillas del tablero. Nota: la ficha no "domina" la diagonal (/).

Problema

Perpendicular común a dos rectas en el espacio

Enviado por jmd el 10 de Diciembre de 2011 - 15:34.

Sean $r$ y $s$ dos rectas ortogonales y que no están en el mismo plano. Sea $AB$ su perpendicular común, donde $A$ pertenece a $r$ y $B$ a $s$. Se considera la esfera de diámetro $AB$. Los puntos $M$, de la recta $r$ y $N$, de la recta $s$, son variables, con la condición de que $MN$ sea tangente a la esfera en un punto $T$. Determine el lugar geométrico de $T$. Nota: el plano que contiene a $B$ y $r$ es perpendicular a $s$.

Problema

Condiciones extravagantes para n+1 números

Enviado por jmd el 10 de Diciembre de 2011 - 15:32.

Sea $n$ un número entero mayor que 1. Determine los números reales $x_1, x_2,\ldots, x_n\leq 1$ y $x_{n+1}>0$, que verifiquen las dos condiciones siguientes:
$$\sqrt{x_1}+\sqrt[3]{x_2}+\ldots+\sqrt[n-1]{x_n}=n\sqrt[2]{x_{n+1}}$$
$$\frac{x_1+x_2+ \ldots +x_n}{n}=x_{n+1}$$

Problema

Para entender la pregunta primero tienes que responderla

Enviado por jmd el 10 de Diciembre de 2011 - 15:27.

Determine los posibles valores de la suma de los digitos de todos los cuadrados perfectos.

Problema

Si le entiendes al enunciado obtienes un punto

Enviado por jmd el 10 de Diciembre de 2011 - 14:20.

Demostrar que todo número natural $n\leq 2^{1000000}$ puede ser obtenido a partir de 1 haciendo menos de 1100000 sumas; más precisamente: que hay una sucesión finita de números naturales $x_0, x_1,\ldots,x_k$, con $k < 1100000$, $x_0 = 1, x_k = n$ tal que para cada $i = 1, 2,\ldots, k$, existen $r, s$ con $0\leq r < i, 0 \leq s < i$, y $x_i = x_r + x_s$.

Problema

Eliges, sumas, y te vas...

Enviado por jmd el 10 de Diciembre de 2011 - 14:18.

Sean $n, r$ dos enteros positivos. Se desea construir $r$ subconjuntos $A_1, A_2,\ldots, A_r$ de $\{0, 1,\ldots, n-1\}$ cada uno de ellos con exactamente $k$ elementos y tales que, para cada entero $x$, $0\leq x \leq n-1$, existen $x_1$ en $A_1$, $x_2$ en $A_2$ ,... , $x_r$ en $A_r$ (un elemento en cada conjunto) con $x = x_1 + x_2\dots+ x_r$. Hallar el menor valor posible de $k$ en función de $n$ y $r$.

Problema

Transformación de acutángulo a equilátero (en el circuncírculo de aquél)

Enviado por jmd el 10 de Diciembre de 2011 - 14:16.

Se dan los puntos $A, B, C$ sobre una circunferencia $K$ de manera que el triángulo $ABC$ sea acutángulo. Sea $P$ un punto interior a $K$. Se trazan las rectas $AP, BP, CP$, que cortan de nuevo a la circunferencia en $X, Y, Z$. Determinar el punto $P$ que hace equilátero al triángulo $XYZ$.

Problema

Tablero lampareado

Enviado por jmd el 10 de Diciembre de 2011 - 14:14.

En cada casilla de un tablero $n\times n$ hay una lámpara. Al ser tocada una lámpara, cambian de estado ella misma y todas las lámparas situadas en la fila y la columna que ella determina (las que están encendidas se apagan y las apagadas se encienden). Inicialmente todas están apagadas. Demostrar que siempre es posible, con una sucesión adecuada de toques, lograr que todo el tablero quede encendido y encontrar, en función de $n$, el número mínimo de toques para que se enciendan todas las lámparas.